Med Sure II Exam 1

Ace your homework & exams now with Quizwiz!

During a physical examination of an older patient, the nurse palpates the point of maximal impulse (PMI) in the sixth intercostal space lateral to the left midclavicular line. The best follow-up action for the nurse to take will be to A. ask about risk factors for atherosclerosis. B. determine family history of heart disease. C. assess for symptoms of left ventricular hypertrophy. D. auscultate carotid arteries for the presence of a bruit.

C. assess for symptoms of left ventricular hypertrophy.

When titrating IV nitroglycerin for a patient with a myocardial infarction (MI), which action will the nurse take to evaluate the effectiveness of the drug? a. Monitor heart rate. c. Check blood pressure. b. Ask about chest pain. d. Observe for dysrhythmias.

b. Ask about chest pain.

A patient comes to the clinic complaining of frequent, watery stools for the past 2 days. Which action should the nurse take first? a. Obtain the baseline weight. b. Check the patient's blood pressure. c. Draw blood for serum electrolyte levels. d. Ask about extremity numbness or tingling.

b. Check the patient's blood pressure.

The nurse is admitting a patient newly diagnosed with peripheral artery disease. Which admission order should the nurse question? a. Cilostazol drug therapy b. Omeprazole drug therapy c. Use of treadmill for exercise d. Exercise to the point of discomfort

b. Omeprazole drug therapy

A patient whose heart monitor shows sinus tachycardia, rate 132, is apneic, and has no palpable pulses. What action should the nurse take next? a. Perform synchronized cardioversion. b. Start cardiopulmonary resuscitation (CPR). c. Give atropine per agency dysrhythmia protocol. d. Provide supplemental O2 via non-rebreather mask.

b. Start cardiopulmonary resuscitation (CPR)

Which assessment finding in a patient who is admitted with infective endocarditis (IE) is most important to communicate to the health care provider? a. Generalized muscle aching b. Sudden onset right flank pain c. Janeway's lesions on the palms d. Temperature 100.7°F (38.1°C)

b. Sudden onset right flank pain

A patient who is being admitted to the emergency department with intermittent chest pain gives the following list of daily medications to the nurse. Which medication has the most immediate implications for the patient's care? a. captopril c. furosemide (Lasix) b. sildenafil (Viagra) d. warfarin (Coumadin)

b. sildenafil (Viagra)

A patient has a 6-cm thoracic aortic aneurysm that was discovered during routine chest x-ray. When obtaining an admission history from the patient, it will be most important for the nurse to ask about a. low back pain. b. trouble swallowing. c. abdominal tenderness. d. changes in bowel habits.

b. trouble swallowing.

When assessing a newly admitted patient, the nurse notes a murmur along the left sternal border. To acquire more information about the murmur, which action will the nurse take? a.Palpate the peripheral pulses. b. Determine the timing of the sound. c. Find the point of maximal impulse. d. Compare apical and radial pulse rates.

b. Determine the timing of the sound.

A patient has a junctional escape rhythm on the monitor. The nurse will expect the patient to have a heart rate of _____ beats/min. a. 15 to 20 c. 40 to 60 b. 20 to 40 d. 60 to 100

c. 40 to 60

Which action will the nurse implement for a patient who arrives for a calcium-scoring CT scan? a. Insert an IV catheter. b. Administer oral sedative medications. c. Teach the patient about the procedure. d. Confirm that the patient has been fasting.

c. Teach the patient about the procedure.

To determine the effects of therapy for a patient who is being treated for heart failure, which laboratory test result will the nurse plan to review? a.Troponin b. Homocysteine (Hcy) c. Low-density lipoprotein (LDL) d. B-type natriuretic peptide (BNP)

d. B-type natriuretic peptide (BNP)

The nurse is reviewing the laboratory results for newly admitted patients on the cardiovascular unit. Which laboratory result is most important to communicate as soon as possible to the health care provider? a. High troponin I level b. Increased triglyceride level c. Very low homocysteine level d. Elevated high-sensitivity C-reactive protein level

a. High troponin I level

A patient with a venous thromboembolism (VTE) is started on enoxaparin (Lovenox) and warfarin (Coumadin). The patient asks the nurse why two medications are necessary. Which response by the nurse is most accurate? a. "Taking two blood thinners greatly reduces the risk for another clot to form." b. "Enoxaparin will work right away, but warfarin takes several days to begin preventing clots." c. "Enoxaparin will start to dissolve the clot, and warfarin will prevent any more clots from forming." d. "Because of the risk for a blood clot in the lungs, it is important for you to take more than one blood thinner."

b. "Enoxaparin will work right away, but warfarin takes several days to begin preventing clots."

A registered nurse (RN) is observing a student nurse who is doing a physical assessment on a patient. The RN will need to intervene immediately if the student nurse a. presses on the skin over the tibia for 10 seconds to check for edema. b. palpates both carotid arteries simultaneously to compare pulse quality. c. documents a murmur heard along the right sternal border as a pulmonic murmur. d. places the patient in the left lateral position to check for the point of maximal impulse.

b. palpates both carotid arteries simultaneously to compare pulse quality.

The nurse has received the laboratory results for a patient who developed chest pain 4 hours ago and may be having a myocardial infarction. The laboratory test result most helpful in indicating myocardial damage will be a. myoglobin. b. troponins T and I. c. homocysteine (Hcy) d. creatine kinase-MB (CK-MB).

b. troponins T and I.

Which information obtained by the nurse who is admitting the patient for magnetic resonance imaging (MRI) will be important to report to the health care provider before the MRI? a. The patient has an allergy to shellfish. b. The patient has a history of atherosclerosis. c. The patient has a permanent cardiac pacemaker. d. The patient took the prescribed heart medications today.

c. The patient has a permanent cardiac pacemaker.

A patient is scheduled for a cardiac catheterization with coronary angiography. Before the test, the nurse informs the patient that a. it will be important not to move at all during the procedure. b. monitored anesthesia care will be provided during the procedure. c. a flushed feeling may be noticed when the contrast dye is injected. d. arterial pressure monitoring will be required for 24 hours after the test.

c. a flushed feeling may be noticed when the contrast dye is injected.

Which topic should the nurse include in patient teaching for a patient with a venous stasis ulcer on the left lower leg? a. Need to increase carbohydrate intake b. Methods of keeping the wound area dry c. Purpose of prophylactic antibiotic therapy d. Application of elastic compression stockings

d. Application of elastic compression stockings

A patient develops sinus bradycardia at a rate of 32 beats/min, has a blood pressure (BP) of 80/42 mm Hg, and is complaining of feeling faint. Which action should the nurse take next? a. Recheck the heart rhythm and BP in 5 minutes. b. Have the patient perform the Valsalva maneuver. c. Give the scheduled dose of diltiazem (Cardizem). d. Apply the transcutaneous pacemaker (TCP) pads.

d. Apply the transcutaneous pacemaker (TCP) pads.

The health care provider has prescribed bed rest with the feet elevated for a patient admitted to the hospital with venous thromboembolism. Which action by the nurse to elevate the patient's feet is best? a. The patient is placed in the Trendelenburg position. b. Two pillows are positioned under the affected leg. c. The bed is elevated at the knee and pillows are placed under the feet. d. One pillow is placed under the thighs and two pillows are placed under the lower legs.

d. One pillow is placed under the thighs and two pillows are placed under the lower legs.

To determine whether there is a delay in impulse conduction through the ventricles, the nurse will measure the duration of the patient's a. P wave. c. PR interval. b. Q wave. d. QRS complex.

d. QRS complex.

When analyzing the rhythm of a patient's electrocardiogram (ECG), the nurse will need to investigate further upon finding a(n) a. isoelectric ST segment. b. PR interval of 0.18 second. c. QT interval of 0.38 second. d. QRS interval of 0.14 second.

d. QRS interval of 0.14 second.

When discussing risk factor modification for a patient who has a 5-cm abdominal aortic aneurysm, the nurse will focus teaching on which patient risk factor? a. Male gender c. Abdominal trauma history b. Turner syndrome d. Uncontrolled hypertension

d. Uncontrolled hypertension

Which action could the nurse delegate to unlicensed assistive personnel (UAP) trained as electrocardiogram (ECG) technicians working on the cardiac unit? a. Select the best lead for monitoring a patient with an admission diagnosis of Dressler syndrome. b. Obtain a list of herbal medications used at home while admitting a new patient with pericarditis. c. Teach about the need to monitor the weight daily for a patient who has hypertrophic cardiomyopathy. d. Watch the heart monitor for changes in rhythm while a patient who had a valve replacement ambulates.

d. Watch the heart monitor for changes in rhythm while a patient who had a valve replacement ambulates.

While doing the hospital admission assessment for a thin older adult, the nurse observes pulsation of the abdominal aorta in the epigastric area. Which action should the nurse take next? a. Teach the patient about aneurysms. b. Notify the hospital rapid response team. c. Instruct the patient to remain on bed rest. d. Document the finding in the patient chart.

d. Document the finding in the patient chart.

The nurse notes that a patient who was admitted with heart failure has jugular venous distention (JVD) when lying flat in bed. Which follow-up action should the nurse take next? a.Obtain vital signs, including oxygen saturation. b. Have the patient perform the Valsalva maneuver. c. Document this JVD finding in the patient's record. d. Observe for JVD with the patient elevated 45 degrees.

d. Observe for JVD with the patient elevated 45 degrees.

A 74-yr-old patient has just arrived in the emergency department. After assessment reveals a pulse deficit of 46 beats, the nurse will anticipate that the patient may require A. emergent cardioversion. B. a cardiac catheterization. C. hourly blood pressure (BP) checks. D. electrocardiographic (ECG) monitoring.

d. electrocardiographic (ECG) monitoring.

Which patient statement to the nurse is most consistent with the diagnosis of venous insufficiency? a. "I can't get my shoes on at the end of the day." b. "I can't ever seem to get my feet warm enough." c. "I have burning leg pains after I walk two blocks." d. "I wake up during the night because my legs hurt."

a. "I can't get my shoes on at the end of the day."

A few days after experiencing a myocardial infarction (MI) and successful percutaneous coronary intervention, the patient states, "It was just a little chest pain. As soon as I get out of here, I'm going for my vacation as planned." Which reply would be most appropriate for the nurse to make? a. "What do you think caused your chest pain?" b. "Where are you planning to go for your vacation?" c. "Sometimes plans need to change after a heart attack." d. "Recovery from a heart attack takes at least a few weeks."

a. "What do you think caused your chest pain?"

A patient with rheumatic fever has subcutaneous nodules, erythema marginatum, and polyarthritis. The patient reports that discomfort in the joints prevents favorite activities such as taking a daily walk and working on sewing projects. Based on these findings, which nursing diagnosis statement would be appropriate? a. Activity intolerance related to arthralgia b. Anxiety related to permanent joint fixation c. Altered body image related to polyarthritis d. Social isolation related to pain and swelling

a. Activity intolerance related to arthralgia

When preparing to defibrillate a patient, in which order will the nurse perform the following steps? (Put a comma and a space between each answer choice [A, B, C, D, E].) a. Turn the defibrillator on. b. Deliver the electrical charge. c. Select the appropriate energy level. d. Place the hands-free, multifunction defibrillator pads on the patient's chest. e. Check the location of other staff and call out "all clear."

ANS: A, C, D, E, B

A 20-yr-old patient has a mandatory electrocardiogram (ECG) before participating on a college soccer team and is found to have sinus bradycardia, rate 52. Blood pressure (BP) is 114/54 mm Hg, and the student denies any health problems. What action by the nurse is most appropriate? a. Allow the student to participate on the soccer team. b. Refer the student to a cardiologist for further testing. c. Tell the student to stop playing immediately if any dyspnea occurs. d. Obtain more detailed information about the student's family health history.

a. Allow the student to participate on the soccer team.

When planning care for a patient hospitalized with a streptococcal infective endocarditis (IE), which intervention is most appropriate for the nurse to include? a. Arrange for placement of a long-term IV catheter. b. Monitor labs for levels of streptococcal antibodies. c. Teach the importance of completing all oral antibiotics. d. Encourage the patient to begin regular aerobic exercise.

a. Arrange for placement of a long-term IV catheter.

When admitting a patient with a non-ST-segment-elevation myocardial infarction (NSTEMI) to the intensive care unit, which action should the nurse perform first? a. Attach the heart monitor. c. Assess the peripheral pulses. b. Obtain the blood pressure. d. Auscultate the breath sounds.

a. Attach the heart monitor.

Two days after an acute myocardial infarction (MI), a patient complains of stabbing chest pain that increases with a deep breath. Which action will the nurse take first? a. Auscultate the heart sounds. b. Check the patient's temperature. c. Give the PRN acetaminophen (Tylenol). d. Notify the patient's health care provider.

a. Auscultate the heart sounds.

The nurse is developing a discharge teaching plan for a patient diagnosed with thromboangiitis obliterans (Buerger's disease). Which expected outcome has the highest priority for this patient? a. Cessation of all tobacco use b. Control of serum lipid levels c. Maintenance of appropriate weight d. Demonstration of meticulous foot care

a. Cessation of all tobacco use

An older patient with a history of an abdominal aortic aneurysm arrives at the emergency department (ED) with severe back pain and absent pedal pulses. Which action should the nurse take first? a. Check the blood pressure. b. Draw blood for laboratory testing. c. Assess for the presence of an abdominal bruit. d. Determine any family history of heart disease.

a. Check the blood pressure.

A patient has a magnesium level of 1.3 mg/dL. Which assessment would help the nurse identify a likely cause of this value? a. Daily alcohol intake b. Dietary protein intake c. Multivitamin/mineral use d. Over-the-counter (OTC) laxative use

a. Daily alcohol intake

A patient who has been receiving diuretic therapy is admitted to the emergency department with a serum potassium level of 3.0 mEq/L. The nurse should alert the health care provider immediately that the patient is on which medication? a. Digoxin (Lanoxin) 0.25 mg/day b. Metoprolol (Lopressor) 12.5 mg/day c. Ibuprofen (Motrin) 400 mg every 6 hours d. Lantus insulin 24 U subcutaneously every evening

a. Digoxin (Lanoxin) 0.25 mg/day

Which action can the registered nurse (RN) who is caring for a critically ill patient with multiple IV lines and medications delegate to a licensed practical/vocational nurse (LPN/LVN)? a. Flush a saline lock with normal saline. b. Verify blood products prior to administration. c. Remove the patient's central venous catheter. d. Titrate the flow rate of vasoactive IV medications.

a. Flush a saline lock with normal saline.

A patient who has recently started taking pravastatin (Pravachol) and niacin reports several symptoms to the nurse. Which information is most important to communicate to the health care provider? a. Generalized muscle aches and pains b. Dizziness with rapid position changes c. Nausea when taking the drugs before meals d. Flushing and pruritus after taking the drugs

a. Generalized muscle aches and pains

When caring for a patient who has just arrived on the telemetry unit after having cardiac catheterization, which nursing intervention should the nurse delegate to a licensed practical/vocational nurse (LPN/LVN)? a. Give the scheduled aspirin and lipid-lowering medication. b. Perform the initial assessment of the catheter insertion site. c. Teach the patient about the usual postprocedure plan of care. d. Titrate the heparin infusion according to the agency protocol.

a. Give the scheduled aspirin and lipid-lowering medication.

After the nurse gives IV atropine to a patient with symptomatic type 1, second-degree atrioventricular (AV) block, which finding indicates that the drug has been effective? a. Increase in the patient's heart rate b. Increase in strength of peripheral pulses c. Decrease in premature atrial contractions d. Decrease in premature ventricular contractions

a. Increase in the patient's heart rate

A postoperative patient who had surgery for a perforated gastric ulcer has been receiving nasogastric suction for 3 days. The patient now has a serum sodium level of 127 mEq/L (127 mmol/L). Which prescribed therapy should the nurse question? a. Infuse 5% dextrose in water at 125 mL/hr. b. Administer 3% saline at 50 mL/hr for a total of 200 mL. c. Administer IV morphine sulfate 4 mg every 2 hours PRN. d. Give IV metoclopramide (Reglan) 10 mg every 6 hours PRN for nausea.

a. Infuse 5% dextrose in water at 125 mL/hr.

A patient receives 3% NaCl solution for correction of hyponatremia. Which assessment is most important for the nurse to monitor for while the patient is receiving this infusion? a. Lung sounds c. Peripheral pulses b. Urinary output d. Peripheral edema

a. Lung sounds

A patient who is lethargic and exhibits deep, rapid respirations has the following arterial blood gas (ABG) results: pH 7.32, PaO2 88 mm Hg, PaCO2 37 mm Hg, and HCO3 16 mEq/L. How should the nurse interpret these results? a. Metabolic acidosis c. Respiratory acidosis b. Metabolic alkalosis d. Respiratory alkalosis

a. Metabolic acidosis

Which action should the nurse include in the plan of care when caring for a patient admitted with acute decompensated heart failure (ADHF) who is receiving nesiritide (Natrecor)? a. Monitor blood pressure frequently. b. Encourage patient to ambulate in room. c. Titrate nesiritide slowly before stopping. d. Teach patient about home use of the drug.

a. Monitor blood pressure frequently.

The nurse notes a serum calcium level of 7.9 mg/dL for a patient who has chronic malnutrition. Which action should the nurse take next? a. Monitor ionized calcium level. b. Give oral calcium citrate tablets. c. Check parathyroid hormone level. d. Administer vitamin D supplements.

a. Monitor ionized calcium level.

A patient with renal failure who arrives for outpatient hemodialysis is unresponsive to questions and has decreased deep tendon reflexes. Family members report that the patient has been taking aluminum hydroxide/magnesium hydroxide suspension (Maalox) at home for indigestion. Which action should the nurse take first? a. Notify the patient's health care provider. b. Obtain an order to draw a potassium level. c. Review the last magnesium level on the patient's chart. d. Teach the patient about magnesium-containing antacids.

a. Notify the patient's health care provider

The nurse is caring for a patient immediately after repair of an abdominal aortic aneurysm. On assessment, the patient has absent popliteal, posterior tibial, and dorsalis pedis pulses. The legs are cool and mottled. Which action should the nurse take first? a. Notify the surgeon and anesthesiologist. b. Wrap both the legs in a warming blanket. c. Document the findings and recheck in 15 minutes. d. Compare findings to the preoperative assessment of the pulses.

a. Notify the surgeon and anesthesiologist.

Which assessment finding in a patient admitted with acute decompensated heart failure (ADHF) requires the most immediate action by the nurse? a. O2 saturation of 88% c. Heart rate of 106 beats/min b. Weight gain of 1 kg (2.2 lb) d. Urine output of 50 mL over 2 hours

a. O2 saturation of 88%

The nurse is caring for a patient with critical limb ischemia who has just arrived on the nursing unit after having percutaneous transluminal balloon angioplasty. Which action should the nurse perform first? a. Obtain vital signs. c. Assess pedal pulses. b. Teach wound care. d. Check the wound site.

a. Obtain vital signs.

Following a thyroidectomy, a patient complains of "a tingling feeling around my mouth." Which assessment should the nurse complete? a. Presence of the Chvostek's sign b. Abnormal serum potassium level c. Decreased thyroid hormone level d. Bleeding on the patient's dressing

a. Presence of the Chvostek's sign

When caring for a patient with renal failure on a low phosphate diet, the nurse will inform unlicensed assistive personnel (UAP) to remove which food from the patient's food tray? a. Skim milk c. Mixed green salad b. Grape juice d. Fried chicken breast

a. Skim milk

A patient in the outpatient clinic has a new diagnosis of peripheral artery disease (PAD). Which group of drugs will the nurse plan to include when teaching about PAD management? a. Statins b. Antibiotics c. Thrombolytics d. Anticoagulants

a. Statins

A patient who is 2 days post femoral popliteal bypass graft to the right leg is being cared for on the vascular unit. Which action by a licensed practical/vocational nurse (LPN/LVN) caring for the patient requires the registered nurse (RN) to intervene? a. The LPN/LVN has the patient to sit in a chair for 2 hours. b. The LPN/LVN gives the prescribed aspirin after breakfast. c. The LPN/LVN assists the patient to walk 40 feet in the hallway. d. The LPN/LVN places the patient in Fowler's position for meals.

a. The LPN/LVN has the patient to sit in a chair for 2 hours.

The nurse is caring for a patient with aortic stenosis. Which assessment data obtained by the nurse would be most important to report to the health care provider? a. The patient complains of chest pressure when ambulating. b. A loud systolic murmur is heard along the right sternal border. c. A thrill is palpated at the second intercostal space, right sternal border. d. The point of maximum impulse (PMI) is at the left midclavicular line.

a. The patient complains of chest pressure when ambulating.

After teaching a patient with newly diagnosed Raynaud's phenomenon about how to manage the condition, which action by the patient best demonstrates that the teaching has been effective? a. The patient exercises indoors during the winter months. b. The patient immerses hands in hot water when they turn pale. c. The patient takes pseudoephedrine (Sudafed) for cold symptoms. d. The patient avoids taking nonsteroidal antiinflammatory drugs (NSAIDs).

a. The patient exercises indoors during the winter months.

A patient has a serum calcium level of 7.0 mEq/L. Which assessment finding is most important for the nurse to report to the health care provider? a. The patient is experiencing laryngeal stridor. b. The patient complains of generalized fatigue. c. The patient's bowels have not moved for 4 days. d. The patient has numbness and tingling of the lips.

a. The patient is experiencing laryngeal stridor.

During a visit to a 78-yr-old patient with chronic heart failure, the home care nurse finds that the patient has ankle edema, a 2-kg weight gain over the past 2 days, and complains of "feeling too tired to get out of bed." Based on these data, a correct nursing diagnosis for the patient is a. activity intolerance related to fatigue. b. impaired skin integrity related to edema. c. disturbed body image related to weight gain. d. impaired gas exchange related to dyspnea on exertion.

a. activity intolerance related to fatigue.

A patient with dilated cardiomyopathy has new onset atrial fibrillation that has been unresponsive to drug therapy for several days. Teaching for this patient would include information about a. anticoagulant therapy. c. emergency cardioversion. b. permanent pacemakers. d. IV adenosine (Adenocard).

a. anticoagulant therapy.

A patient in the intensive care unit with acute decompensated heart failure (ADHF) complains of severe dyspnea and is anxious, tachypneic, and tachycardic. Several drugs have been ordered for the patient. The nurse's priority action will be to a. give PRN IV morphine sulfate 4 mg. b. give PRN IV diazepam (Valium) 2.5 mg. c. increase nitroglycerin infusion by 5 mcg/min. d. increase dopamine infusion by 2 mcg/kg/min.

a. give PRN IV morphine sulfate 4 mg.

While caring for a patient with aortic stenosis, the nurse identifies a nursing diagnosis of acute pain related to decreased coronary blood flow. An appropriate nursing intervention for this patient would be to a. promote rest to decrease myocardial oxygen demand. b. teach the patient about the need for anticoagulant therapy. c. teach the patient to use sublingual nitroglycerin for chest pain. d. raise the head of the bed 60 degrees to decrease venous return.

a. promote rest to decrease myocardial oxygen demand.

Three days after experiencing a myocardial infarction (MI), a patient who is scheduled for discharge asks for assistance with hygiene activities, saying, "I am too nervous about my heart to be alone while I get washed up." Based on this information, which nursing diagnosis is appropriate? a. Activity intolerance related to weakness b. Anxiety related to change in health status c. Denial related to lack of acceptance of the MI d. Altered body image related to cardiac disease

b. Anxiety related to change in health status

When evaluating the discharge teaching for a patient with chronic peripheral artery disease (PAD), the nurse determines a need for further instruction when the patient says, "I will a. use a heating pad on my feet at night to increase the circulation." b. buy some loose clothes that do not bind across my legs or waist." c. walk to the point of pain, rest, and walk again for at least 30 minutes 3 times a week." d. change my position every hour and avoid long periods of sitting with my legs crossed."

a. use a heating pad on my feet at night to increase the circulation."

The nurse is caring for a patient who was admitted to the coronary care unit following an acute myocardial infarction (AMI) and percutaneous coronary intervention the previous day. Teaching for this patient would include a. when cardiac rehabilitation will begin. b. the typical emotional responses to AMI. c. information regarding discharge medications. d. the pathophysiology of coronary artery disease.

a. when cardiac rehabilitation will begin.

After receiving change-of-shift report on four patients admitted to a heart failure unit, which patient should the nurse assess first? a. A patient who reported dizziness after receiving the first dose of captopril b. A patient who is cool and clammy, with new-onset confusion and restlessness c. A patient who has crackles bilaterally in the lung bases and is receiving oxygen. d. A patient who is receiving IV nesiritide (Natrecor) and has a blood pressure of 100/62

b. A patient who is cool and clammy, with new-onset confusion and restlessness

The nurse hears a murmur between the S1 and S2 heart sounds at the patient's left fifth intercostal space and midclavicular line. How will the nurse record this information? a. Systolic murmur heard at mitral area b. Systolic murmur heard at Erb's point c. Diastolic murmur heard at aortic area d. Diastolic murmur heard at the point of maximal impulse

a. Systolic murmur heard at mitral area

The standard policy on the cardiac unit states, "Notify the health care provider for mean arterial pressure (MAP) less than 70 mm Hg." The nurse will need to call the health care provider about the a. postoperative patient with a BP of 116/42 mm Hg. b. newly admitted patient with a BP of 150/87 mm Hg. c. patient with left ventricular failure who has a BP of 110/70 mm Hg. d. patient with a myocardial infarction who has a BP of 140/86 mm Hg.

a. postoperative patient with a BP of 116/42 mm Hg.

The nurse is admitting a patient with possible rheumatic fever. Which question on the admission health history focuses on a pertinent risk factor for rheumatic fever? a. "Do you use any illegal IV drugs?" b. "Have you had a recent sore throat?" c. "Have you injured your chest in the last few weeks?" d. "Do you have a family history of congenital heart disease?"

b. "Have you had a recent sore throat?"

The nurse has started discharge teaching for a patient who is to continue warfarin (Coumadin) after hospitalization for venous thromboembolism (VTE). The nurse determines that additional teaching is needed when the patient says which of the following? a. "I should get a Medic Alert device stating that I take warfarin." b. "I should reduce the amount of green, leafy vegetables that I eat." c. "I will need routine blood tests to monitor the effects of the warfarin." d. "I will check with my health care provider before I begin any new drugs."

b. "I should reduce the amount of green, leafy vegetables that I eat."

When evaluating the effectiveness of preoperative teaching with a patient scheduled for coronary artery bypass graft (CABG) surgery using the internal mammary artery, the nurse determines that additional teaching is needed when the patient says which of the following? a. "They will circulate my blood with a machine during surgery." b. "I will have incisions in my leg where they will remove the vein." c. "They will use an artery near my heart to go around the area that is blocked." d. "I will need to take an aspirin every day after the surgery to keep the graft open."

b. "I will have incisions in my leg where they will remove the vein."

The home health nurse cares for an alert and oriented older adult patient with a history of dehydration. Which instructions should the nurse give this patient related to fluid intake? a. "Drink more fluids in the late evening." b. "Increase fluids if your mouth feels dry." c. "More fluids are needed if you feel thirsty." d. "If you feel confused, you need more to drink."

b. "Increase fluids if your mouth feels dry."

After the nurse teaches the patient about the use of carvedilol (Coreg) in preventing anginal episodes, which statement by a patient indicates that the teaching has been effective? a. "Carvedilol will help my heart muscle work harder." b. "It is important not to suddenly stop taking the carvedilol." c. "I can expect to feel short of breath when taking carvedilol." d. "Carvedilol will increase the blood flow to my heart muscle."

b. "It is important not to suddenly stop taking the carvedilol."

Which instructions should the nurse include in a teaching plan for an older patient newly diagnosed with peripheral artery disease (PAD)? a. "Exercise only if you do not experience any pain." b. "It is very important that you stop smoking cigarettes." c. "Try to keep your legs elevated whenever you are sitting." d. "Put elastic compression stockings on early in the morning."

b. "It is very important that you stop smoking cigarettes."

While working in the outpatient clinic, the nurse notes that a patient has a history of intermittent claudication. Which statement by the patient would support this information? a. "When I stand too long, my feet start to swell." b. "My legs cramp when I walk more than a block." c. "I get short of breath when I climb a lot of stairs." d. "My fingers hurt when I go outside in cold weather."

b. "My legs cramp when I walk more than a block."

A patient who is recovering from an acute myocardial infarction (AMI) asks the nurse when sexual intercourse can be resumed. Which response by the nurse is best? a. "Most patients are able to enjoy intercourse without any complications." b. "Sexual activity uses about as much energy as climbing two flights of stairs." c. "The doctor will provide sexual guidelines when your heart is strong enough." d. "Holding and cuddling are good ways to maintain intimacy after a heart attack."

b. "Sexual activity uses about as much energy as climbing two flights of stairs."

A patient has a parenteral nutrition infusion of 25% dextrose. A student nurse asks the nurse why a peripherally inserted central catheter was inserted. Which response by the nurse is accurate? a. "The prescribed infusion can be given more rapidly when the patient has a central line." b. "The hypertonic solution will be more rapidly diluted when given through a central line." c. "There is a decreased risk for infection when 25% dextrose is infused through a central line." d. "The required blood glucose monitoring is based on samples obtained from a central line."

b. "The hypertonic solution will be more rapidly diluted when given through a central line."

After receiving change of shift report, which patient admitted to the emergency department should the nurse assess first? a. A 67-yr-old patient who has a gangrenous left foot ulcer with a weak pedal pulse b. A 50-yr-old patient who is complaining of sudden sharp and severe upper back pain c. A 39-yr-old patient who has right calf tenderness, redness, and swelling after a plane ride d. A 58-yr-old patient who is taking anticoagulants for atrial fibrillation and has black stools

b. A 50-yr-old patient who is complaining of sudden sharp and severe upper back pain

When the nurse is screening patients for possible peripheral arterial disease, indicate where the posterior tibial artery will be palpated. a. 1 b. 2 c. 3 d. 4

b. 2

Which actions could the nurse delegate to unlicensed assistive personnel (UAP) who are providing care for a patient who is at risk for venous thromboembolism? a. Monitor for any bleeding after anticoagulation therapy is started. b. Apply sequential compression device whenever the patient is in bed. c. Ask the patient about use of herbal medicines or dietary supplements. d. Instruct the patient to call immediately if any shortness of breath occurs.

b. Apply sequential compression device whenever the patient is in bed.

A patient with new-onset confusion and hyponatremia is being admitted. When making room assignments, the charge nurse should take which action? a. Assign the patient to a semi-private room. b. Assign the patient to a room near the nurse's station. c. Place the patient in a room nearest to the water fountain. d. Place the patient on telemetry to monitor for peaked T waves..

b. Assign the patient to a room near the nurse's station.

After reviewing a patient's history, vital signs, physical assessment, and laboratory data, which information shown in the accompanying figure is most important for the nurse to communicate to the health care provider? a. Hyperglycemia b. Bilateral crackles c. Q waves on ECG d. Elevated troponin

b. Bilateral crackles

A patient with multiple draining wounds is admitted for hypovolemia. Which assessment would be the most accurate way for the nurse to evaluate fluid balance? a. Skin turgor b. Daily weight c. Urine output d. Edema presence

b. Daily weight

To improve the physical activity level for a mildly obese 71-yr-old patient, which action should the nurse plan to take? a. Stress that weight loss is a major benefit of increased exercise. b. Determine what kind of physical activities the patient usually enjoys. c. Tell the patient that older adults should exercise for no more than 20 minutes at a time. d. Teach the patient to include a short warm-up period at the beginning of physical activity.

b. Determine what kind of physical activities the patient usually enjoys.

An older adult patient who is malnourished presents to the emergency department with a serum protein level of 5.2 g/dL. The nurse would expect which clinical manifestation? a. Pallor b. Edema c. Confusion d. Restlessness

b. Edema

The nurse notes that a patient who was admitted with diabetic ketoacidosis has rapid, deep respirations. Which action should the nurse take? a. Give the prescribed PRN lorazepam (Ativan). b. Encourage the patient to take deep slow breaths. c. Start the prescribed PRN oxygen at 2 to 4 L/min. d. Administer the prescribed normal saline bolus and insulin.

b. Encourage the patient to take deep slow breaths.

Which action should the nurse perform when preparing a patient with supraventricular tachycardia for cardioversion who is alert and has a blood pressure of 110/66 mm Hg? a. Turn the synchronizer switch to the "off" position. b. Give a sedative before cardioversion is implemented. c. Set the defibrillator/cardioverter energy to 360 joules. d. Provide assisted ventilations with a bag-valve-mask device.

b. Give a sedative before cardioversion is implemented.

A patient who had a transverse colectomy for diverticulosis 18 hours ago has nasogastric suction. The patient complains of anxiety and incisional pain. The patient's respiratory rate is 32 breaths/min, and the arterial blood gases (ABGs) indicate respiratory alkalosis. Which action should the nurse take first? a. Check to make sure the nasogastric tube is patent. b. Give the patient the PRN IV morphine sulfate 4 mg. c. Notify the health care provider about the ABG results. d. Teach the patient how to take slow, deep breaths when anxious.

b. Give the patient the PRN IV morphine sulfate 4 mg.

After an acute myocardial infarction (AMI), a patient ambulates in the hospital hallway. When the nurse evaluates the patient's response to the activity, which data would indicate that the exercise level should be decreased? a. O2 saturation drops from 99% to 95%. b. Heart rate increases from 66 to 98 beats/min. c. Respiratory rate goes from 14 to 20 breaths/min. d. Blood pressure (BP) changes from 118/60 to 126/68 mm Hg.

b. Heart rate increases from 66 to 98 beats/min.

A patient who has had chest pain for several hours is admitted with a diagnosis of rule out acute myocardial infarction (AMI). Which laboratory test should the nurse monitor to best determine whether the patient has had an AMI? a. Myoglobin b. Homocysteine c. C-reactive protein d. Cardiac-specific troponin

b. Homocysteine

IV potassium chloride (KCl) 60 mEq is prescribed for treatment of a patient with severe hypokalemia. Which action should the nurse take? a. Administer the KCl as a rapid IV bolus. b. Infuse the KCl at a rate of 10 mEq/hour. c. Only give the KCl through a central venous line. d. Discontinue cardiac monitoring during the infusion.

b. Infuse the KCl at a rate of 10 mEq/hour.

Which action by a new registered nurse (RN) who is orienting to the telemetry unit indicates a good understanding of the treatment of heart dysrhythmias? a. Prepares defibrillator settings at 360 joules for a patient whose monitor shows asystole. b. Injects IV adenosine (Adenocard) over 2 seconds to a patient with supraventricular tachycardia c. Turns the synchronizer switch to the "on" position before defibrillating a patient with ventricular fibrillation d. Gives the prescribed dose of diltiazem (Cardizem) to a patient with new-onset type II second degree AV block

b. Injects IV adenosine (Adenocard) over 2 seconds to a patient with supraventricular tachycardia

Which action will the nurse include in the plan of care for a patient who was admitted with syncopal episodes of unknown origin? a. Explain the association between dysrhythmias and syncope. b. Instruct the patient to call for assistance before getting out of bed. c. Teach the patient about the need to avoid caffeine and other stimulants. d. Tell the patient about the benefits of implantable cardioverter-defibrillators.

b. Instruct the patient to call for assistance before getting out of bed.

The nurse is assessing a patient with myocarditis before giving the scheduled dose of digoxin (Lanoxin). Which finding is most important for the nurse to communicate to the health care provider? a. Leukocytosis b. Irregular pulse c. Generalized myalgia d. Complaint of fatigue

b. Irregular pulse

Which nursing action should be included in the plan of care after endovascular repair of an abdominal aortic aneurysm? a. Record hourly chest tube drainage. b. Monitor fluid intake and urine output. c. Assess the abdominal incision for redness. d. Teach the patient to plan for a long recovery period.

b. Monitor fluid intake and urine output.

Which nursing action can the registered nurse (RN) delegate to experienced unlicensed assistive personnel (UAP) working as telemetry technicians on the cardiac care unit? a. Decide whether a patient's heart rate of 116 requires urgent treatment. b. Observe heart rhythms for multiple patients who have telemetry monitoring. c. Monitor a patient's level of consciousness during synchronized cardioversion. d. Select the best lead for monitoring a patient admitted with acute coronary syndrome.

b. Observe heart rhythms for multiple patients who have telemetry monitoring.

Which admission order written by the health care provider for a patient admitted with infective endocarditis (IE) and a fever would be a priority for the nurse to implement? a. Administer ceftriaxone 1 g IV. b. Order blood cultures drawn from two sites. c. Give acetaminophen (Tylenol) PRN for fever. d. Arrange for a transesophageal echocardiogram.

b. Order blood cultures drawn from two sites.

Which assessment finding by the nurse caring for a patient who has had coronary artery bypass grafting using a right radial artery graft is most important to communicate to the health care provider? a. Complaints of incisional chest pain b. Pallor and weakness of the right hand c. Fine crackles heard at both lung bases d. Redness on both sides of the sternal incision

b. Pallor and weakness of the right hand

After receiving information about four patients during change-of-shift report, which patient should the nurse assess first? a. Patient with acute pericarditis who has a pericardial friction rub b. Patient who has just returned to the unit after balloon valvuloplasty c. Patient who has hypertrophic cardiomyopathy and a heart rate of 116 d. Patient with a mitral valve replacement who has an anticoagulant scheduled

b. Patient who has just returned to the unit after balloon valvuloplasty

After receiving change-of-shift report on a heart failure unit, which patient should the nurse assess first? a. Patient who is taking carvedilol (Coreg) and has a heart rate of 58 b. Patient who is taking digoxin and has a potassium level of 3.1 mEq/L c. Patient who is taking captopril and has a frequent nonproductive cough d. Patient who is taking isosorbide dinitrate/hydralazine (BiDil) and has a headache

b. Patient who is taking digoxin and has a potassium level of 3.1 mEq/L

The nurse who works in the vascular clinic has several patients with venous insufficiency scheduled today. Which patient should the nurse assign to an experienced licensed practical/vocational nurse (LPN/LVN)? a. Patient who has been complaining of increased edema and skin changes in the legs b. Patient who needs wound care for a chronic venous stasis ulcer on the right lower leg c. Patient who has a history of venous thromboembolism and is complaining of dyspnea d. Patient who needs teaching about elastic compression stockings for venous insufficiency

b. Patient who needs wound care for a chronic venous stasis ulcer on the right lower leg

After receiving change-of-shift report on four patients, which patient should the nurse assess first? a. Patient with rheumatic fever who has sharp chest pain with a deep breath b. Patient with acute aortic regurgitation whose blood pressure is 86/54 mm Hg c. Patient with infective endocarditis who has a murmur and splinter hemorrhages d. Patient with dilated cardiomyopathy who has bilateral crackles at the lung bases

b. Patient with acute aortic regurgitation whose blood pressure is 86/54 mm Hg

Which patient at the cardiovascular clinic requires the most immediate action by the nurse? a. Patient with type 2 diabetes whose current blood glucose level is 145 mg/dL b. Patient with stable angina whose chest pain has recently increased in frequency c. Patient with familial hypercholesterolemia and a total cholesterol of 465 mg/dL d. Patient with chronic hypertension whose blood pressure today is 172/98 mm Hg

b. Patient with stable angina whose chest pain has recently increased in frequency

A patient's heart monitor shows a pattern of undulations of varying contours and amplitude with no measurable ECG pattern. The patient is unconscious, apneic, and pulseless. Which action should the nurse take first? a. Give epinephrine (Adrenalin) IV. b. Perform immediate defibrillation. c. Prepare for endotracheal intubation. d. Ventilate with a bag-valve-mask device.

b. Perform immediate defibrillation.

A patient is admitted to the emergency department with severe fatigue and confusion. Laboratory studies are done. Which laboratory value will require the most immediate action by the nurse? a. Arterial blood pH is 7.32. b. Serum calcium is 18 mg/dL. c. Serum potassium is 5.1 mEq/L. d. Arterial oxygen saturation is 91%.

b. Serum calcium is 18 mg/dL.

A patient who has a small cell carcinoma of the lung develops syndrome of inappropriate antidiuretic hormone (SIADH). The nurse should notify the health care provider about which assessment finding? a. Serum hematocrit of 42% b. Serum sodium level of 120 mg/dL c. Reported weight gain of 2.2 lb (1 kg) d. Urinary output of 280 mL during past 8 hours

b. Serum sodium level of 120 mg/dL

When developing a community health program to decrease the incidence of rheumatic fever, which action should the community health nurse include? a. Vaccinate high-risk groups in the community with streptococcal vaccine. b. Teach community members to seek treatment for streptococcal pharyngitis. c. Teach about the importance of monitoring temperature when sore throats occur. d. Teach about prophylactic antibiotics to those with a family history of rheumatic fever.

b. Teach community members to seek treatment for streptococcal pharyngitis.

The nurse is admitting a patient who has chest pain. Which assessment data suggest that the pain is caused by an acute myocardial infarction (AMI)? a. The pain increases with deep breathing. b. The pain has lasted longer than 30 minutes. c. The pain is relieved after the patient takes nitroglycerin. d. The pain is reproducible when the patient raises the arms.

b. The pain has lasted longer than 30 minutes.

When assessing a pregnant patient with eclampsia who is receiving IV magnesium sulfate, which finding should the nurse report to the health care provider immediately? a. The bibasilar breath sounds are decreased. b. The patellar and triceps reflexes are absent. c. The patient has been sleeping most of the day. d. The patient reports feeling "sick to my stomach."

b. The patellar and triceps reflexes are absent.

A young adult patient tells the health care provider about experiencing cold, numb fingers when running during the winter, and Raynaud's phenomenon is suspected. The nurse will anticipate teaching the patient about tests for a. hyperglycemia. b. hyperlipidemia. c. autoimmune disorders. d. coronary artery disease.

c. autoimmune disorders.

The nurse is caring for a 64-yr-old patient admitted with mitral valve regurgitation. Which information obtained by the nurse when assessing the patient should be communicated to the health care provider immediately? a. The patient has 4+ peripheral edema. b. The patient has diffuse bilateral crackles. c. The patient has a loud systolic murmur across the precordium. d. The patient has a palpable thrill felt over the left anterior chest.

b. The patient has diffuse bilateral crackles.

A patient is receiving a 3% saline continuous IV infusion for hyponatremia. Which assessment data will require the most rapid response by the nurse? a. The patient's radial pulse is 105 beats/min. b. There are crackles throughout both lung fields. c. There is sediment and blood in the patient's urine. d. The blood pressure increases from 120/80 to 142/94 mm Hg.

b. There are crackles throughout both lung fields.

The nurse identifies the nursing diagnosis of decreased cardiac output related to valvular insufficiency for the patient with infective endocarditis (IE) based on which assessment finding(s)? a. Fever, chills, and diaphoresis b. Urine output less than 30 mL/hr c. Petechiae on the inside of the mouth and conjunctiva d. Increase in heart rate of 15 beats/minute with walking

b. Urine output less than 30 mL/hr

The nurse is caring for a patient who has a central venous access device (CVAD). Which action by the nurse is appropriate? a. Avoid using friction when cleaning around the CVAD insertion site. b. Use the push-pause method to flush the CVAD after giving medications. c. Obtain an order from the health care provider to change CVAD dressing. d. Position the patient's face toward the CVAD during injection cap changes.

b. Use the push-pause method to flush the CVAD after giving medications.

The nurse notes that a patient's heart monitor shows that every other beat is earlier than expected, has no visible P wave, and has a QRS complex that is wide and bizarre in shape. How will the nurse document the rhythm? a. Ventricular couplets b. Ventricular bigeminy c. Ventricular R-on-T phenomenon d. Multifocal premature ventricular contractions

b. Ventricular bigeminy

During the assessment of a young adult patient with infective endocarditis (IE), the nurse would expect to find a. substernal chest pressure. b. a new regurgitant murmur. c. a pruritic rash on the chest. d. involuntary muscle movement.

b. a new regurgitant murmur.

When caring for a patient who is recovering from a sudden cardiac death (SCD) event and has no evidence of an acute myocardial infarction (AMI), the nurse will anticipate teaching the patient that a. sudden cardiac death events rarely reoccur. b. additional diagnostic testing will be required. c. long-term anticoagulation therapy will be needed. d. limiting physical activity will prevent future SCD events.

b. additional diagnostic testing will be required.

To assess the patient with pericarditis for evidence of a pericardial friction rub, the nurse should a. listen for a rumbling, low-pitched, systolic murmur over the left anterior chest. b. auscultate with the diaphragm of the stethoscope on the lower left sternal border. c. ask the patient to cough during auscultation to distinguish the sound from a pleural friction rub. d. feel the precordial area with the palm of the hand to detect vibrations with cardiac contraction.

b. auscultate with the diaphragm of the stethoscope on the lower left sternal border.

The health care provider prescribes an infusion of heparin and daily partial thromboplastin time (PTT) testing for a patient with venous thromboembolism (VTE). The nurse will plan to a. decrease the infusion when the PTT value is 65 seconds. b. avoid giving IM medications to prevent localized bleeding. c. have vitamin K available in case reversal of the heparin is needed. d. monitor posterior tibial and dorsalis pedis pulses with the Doppler.

b. avoid giving IM medications to prevent localized bleeding.

The nurse suspects that the patient with stable angina is experiencing a side effect of the prescribed drug metoprolol (Lopressor) if the a. patient is restless and agitated. b. blood pressure is 90/54 mm Hg. c. patient complains about feeling anxious. d. heart monitor shows normal sinus rhythm.

b. blood pressure is 90/54 mm Hg.

A patient has a sinus rhythm and a heart rate of 72 beats/min. The nurse determines that the PR interval is 0.24 seconds. The most appropriate intervention by the nurse would be to a. notify the health care provider immediately. b. document the finding and monitor the patient. c. give atropine per agency dysrhythmia protocol. d. prepare the patient for temporary pacemaker insertion.

b. document the finding and monitor the patient.

A patient is admitted to the hospital with possible acute pericarditis. The nurse should plan to teach the patient about the purpose of a. blood cultures. b. echocardiography. c. cardiac catheterization. d. 24-hour Holter monitor.

b. echocardiography.

The nurse suspects cardiac tamponade in a patient who has acute pericarditis. To assess for the presence of pulsus paradoxus, the nurse should a. subtract the diastolic blood pressure from the systolic blood pressure. b. note when Korotkoff sounds are auscultated during both inspiration and expiration. c. check the electrocardiogram (ECG) for variations in rate during the respiratory cycle. d. listen for a pericardial friction rub that persists when the patient is instructed to stop breathing.

b. note when Korotkoff sounds are auscultated during both inspiration and expiration.

When teaching the patient with newly diagnosed heart failure about a 2000-mg sodium diet, the nurse explains that foods to be restricted include a. canned and frozen fruits. b.v yogurt and milk products. c. fresh or frozen vegetables. d. eggs and other high-protein foods.

b. yogurt and milk products.

The nurse and unlicensed assistive personnel (UAP) on the telemetry unit are caring for four patients. Which nursing action can be delegated to the UAP? a. Teaching a patient about exercise electrocardiography b. Attaching ECG monitoring electrodes after a patient bathes c. Checking the catheter insertion site for a patient who is recovering from a coronary angiogram d. Monitoring a patient who has just returned to the unit after a transesophageal echocardiogram

b. Attaching ECG monitoring electrodes after a patient bathes

Which nursing intervention is likely to be most effective when assisting the patient with coronary artery disease to make appropriate dietary changes? a. Inform the patient about a diet containing no saturated fat and minimal salt. b. Help the patient modify favorite high-fat recipes by using monounsaturated oils. c. Emphasize the increased risk for heart problems unless the patient makes the dietary changes. d. Give the patient a list of low-sodium, low-cholesterol foods that should be included in the diet.

b. Help the patient modify favorite high-fat recipes by using monounsaturated oils.

A patient has ST segment changes that suggest an acute inferior wall myocardial infarction. Which lead would be best for monitoring the patient? a. I b. II c. V2 d. V6

b. II

When admitting a patient for a cardiac catheterization and coronary angiogram, which information about the patient is most important for the nurse to communicate to the health care provider? a. The patient's pedal pulses are +1. b. The patient is allergic to shellfish. c. The patient had a heart attack 1 year ago. d. The patient has not eaten anything today.

b. The patient is allergic to shellfish.

When auscultating over the patient's abdominal aorta, the nurse hears a loud humming sound. The nurse documents this finding as a a. thrill. b. bruit. c. murmur. d. normal finding.

b. bruit.

When developing a teaching plan for a 61-yr-old patient with multiple risk factors for coronary artery disease (CAD), the nurse should focus primarily on the a. family history of coronary artery disease. b. elevated low-density lipoprotein (LDL) level. c. increased risk associated with the patient's gender. d. increased risk of cardiovascular disease as people age.

b. elevated low-density lipoprotein (LDL) level.

To auscultate for S3 or S4 gallops in the mitral area, the nurse listens with the a.diaphragm of the stethoscope with the patient lying flat. b.bell of the stethoscope with the patient in the left lateral position. c.diaphragm of the stethoscope with the patient in a supine position. d. bell of the stethoscope with the patient sitting and leaning forward.

b.bell of the stethoscope with the patient in the left lateral position.

The nurse is caring for a patient with a massive burn injury and possible hypovolemia. Which assessment data will be of most concern to the nurse? a. Urine output is 30 mL/hr. b. Blood pressure is 90/40 mm Hg. c. Oral fluid intake is 100 mL for the past 8 hours. d. There is prolonged skin tenting over the sternum.

b.v Blood pressure is 90/40 mm Hg.

The home health nurse is visiting a 30-yr-old patient recovering from rheumatic fever without carditis. The nurse establishes the nursing diagnosis of ineffective health maintenance related to lack of knowledge regarding long-term management of rheumatic fever when the patient makes which statement? a. "I will need prophylactic antibiotic therapy for 5 years." b. "I can take aspirin or ibuprofen (Motrin) to relieve my joint pain." c. "I will be immune to future episodes of rheumatic fever after this infection." d. "I should call the health care provider if I am fatigued or have difficulty breathing."

c. "I will be immune to future episodes of rheumatic fever after this infection."

A patient with heart failure has a new order for captopril 12.5 mg PO. After giving the first dose and teaching the patient about the drug, which statement by the patient indicates that teaching has been effective? a. "I will be sure to take the medication with food." b. "I will need to eat more potassium-rich foods in my diet." c. "I will call for help when I need to get up to use the bathroom." d. "I will expect to feel more short of breath for the next few days."

c. "I will call for help when I need to get up to use the bathroom."

In preparation for discharge, the nurse teaches a patient with chronic stable angina how to use the prescribed short-acting and long-acting nitrates. Which patient statement indicates that the teaching has been effective? a. "I will check my pulse rate before I take any nitroglycerin tablets." b. "I will put the nitroglycerin patch on as soon as I get any chest pain." c. "I will stop what I am doing and sit down before I put the nitroglycerin under my tongue." d. "I will be sure to remove the nitroglycerin patch before taking any sublingual nitroglycerin."

c. "I will stop what I am doing and sit down before I put the nitroglycerin under my tongue."

A 53-yr-old patient with stage D heart failure and type 2 diabetes asks the nurse whether heart transplant is a possible therapy. Which response by the nurse is most accurate? a. "Your heart failure has not reached the end stage yet." b. "You could not manage the multiple complications of that surgery." c. "The suitability of a heart transplant for you depends on many factors." d. "Because you have diabetes, you would not be a heart transplant candidate."

c. "The suitability of a heart transplant for you depends on many factors."

A patient with ST-segment elevation in three contiguous electrocardiographic leads is admitted to the emergency department and diagnosed as having an ST-segment-elevation myocardial infarction. Which question should the nurse ask to determine whether the patient is a candidate for thrombolytic therapy? a. "Do you have any allergies?" b. "Do you take aspirin on a daily basis?" c. "What time did your chest pain begin?" d. "Can you rate your chest pain using a 0 to 10 scale?"

c. "What time did your chest pain begin?"

A 19-yr-old student comes to the student health center at the end of the semester complaining that, "My heart is skipping beats." An electrocardiogram (ECG) shows occasional unifocal premature ventricular contractions (PVCs). What action should the nurse take next? a. Insert an IV catheter for emergency use. b. Start supplemental O2 at 2 to 3 L/min via nasal cannula. c. Ask the patient about current stress level and caffeine use. d. Have the patient taken to the nearest emergency department (ED).

c. Ask the patient about current stress level and caffeine use.

Which action by the nurse will determine if the therapies ordered for a patient with chronic constrictive pericarditis are most effective? a. Assess for the presence of a paradoxical pulse. b. Monitor for changes in the patient's sedimentation rate. c. Assess for the presence of jugular venous distention (JVD). d. Check the electrocardiogram (ECG) for ST segment changes.

c. Assess for the presence of jugular venous distention (JVD).

A patient recovering from a myocardial infarction (MI) develops chest pain on day 3 that increases when taking a deep breath and is relieved by leaning forward. Which action should the nurse take as focused follow-up on this symptom? a. Assess the feet for pedal edema. b. Palpate the radial pulses bilaterally. c. Auscultate for a pericardial friction rub. d. Check the heart monitor for dysrhythmias.

c. Auscultate for a pericardial friction rub.

A patient with a history of chronic heart failure is admitted to the emergency department with severe dyspnea and a dry, hacking cough. Which action should the nurse do first? a. Auscultate the abdomen. b. Check the capillary refill. c. Auscultate the breath sounds. d. Ask about the patient's allergies.

c. Auscultate the breath sounds.

Which action should the nurse take first when a patient complains of acute chest pain and dyspnea soon after insertion of a centrally inserted IV catheter? a. Notify the health care provider. b. Offer reassurance to the patient. c. Auscultate the patient's breath sounds. d. Give prescribed PRN morphine sulfate IV.

c. Auscultate the patient's breath sounds.

Which diagnostic test will be most useful to the nurse in determining whether a patient admitted with acute shortness of breath has heart failure? a. Serum troponin b. Arterial blood gases c. B-type natriuretic peptide d. 12-lead electrocardiogram

c. B-type natriuretic peptide

The nurse obtains the following data when assessing a patient who experienced an ST-segment-elevation myocardial infarction (STEMI) 2 days previously. Which information is most important to report to the health care provider? a. The troponin level is elevated. b. The patient denies having a heart attack. c. Bilateral crackles in the mid-lower lobes. d. Occasional premature atrial contractions (PACs).

c. Bilateral crackles in the mid-lower lobes.

The nurse is caring for a patient with a descending aortic dissection. Which assessment finding is most important to report to the health care provider? a. Weak pedal pulses b. Absent bowel sounds c. Blood pressure of 138/88 mm Hg d. 25 mL of urine output over the past hour

c. Blood pressure of 138/88 mm Hg

A patient's heart monitor shows sinus rhythm, rate 64. The PR interval is 0.18 seconds at 1:00 AM, 0.22 seconds at 2:30 PM, and 0.28 seconds at 4:00 PM. Which action should the nurse take next? a. Place the transcutaneous pacemaker pads on the patient. b. Give atropine sulfate 1 mg IV per agency dysrhythmia protocol. c. Call the health care provider before giving scheduled metoprolol (Lopressor). d. Document the patient's rhythm and assess the patient's response to the rhythm.

c. Call the health care provider before giving scheduled metoprolol (Lopressor).

The long-term care nurse is evaluating the effectiveness of protein supplements for an older resident who has a low serum total protein level. Which assessment finding indicates that the patient's condition has improved? a. Hematocrit 28% b. Absence of skin tenting c. Decreased peripheral edema d. Blood pressure 110/72 mm Hg

c. Decreased peripheral edema

After reviewing information shown in the accompanying figure from the medical records of a 43-yr-old patient, which risk factor modification for coronary artery disease should the nurse include in patient teaching? a. Importance of daily physical activity b. Effect of weight loss on blood pressure c. Dietary changes to improve lipid levels d. Cardiac risk associated with previous tobacco use

c. Dietary changes to improve lipid levels

A 46-yr-old service-counter worker undergoes sclerotherapy for treatment of superficial varicose veins at an outpatient center. Which instructions should the nurse provide to the patient before discharge? a. Sitting at the work counter, rather than standing, is recommended. b. Exercise, such as walking or jogging, can cause recurrence of varicosities. c. Elastic compression stockings should be applied before getting out of bed. d. Taking an aspirin daily will help prevent clots from forming around venous valves.

c. Elastic compression stockings should be applied before getting out of bed.

A patient who has chest pain is admitted to the emergency department (ED), and all of the following are ordered. Which one should the nurse arrange to be completed first? a. Chest x-ray b. Troponin level c. Electrocardiogram (ECG) d. Insertion of a peripheral IV

c. Electrocardiogram (ECG)

A patient who is on the telemetry unit develops atrial flutter, rate 150, with associated dyspnea and chest pain. Which action that is included in the hospital dysrhythmia protocol should the nurse do first? a. Obtain a 12-lead electrocardiogram (ECG). b. Notify the health care provider of the change in rhythm. c. Give supplemental O2 at 2 to 3 L/min via nasal cannula. d. Assess the patient's vital signs including O2 saturation.

c. Give supplemental O2 at 2 to 3 L/min via nasal cannula.

A patient with hyperlipidemia has a new order for colesevelam (Welchol). Which nursing action is appropriate when scheduling this medication? a. Administer the medication at the patient's usual bedtime. b. Have the patient take the colesevelam 1 hour before breakfast. A patient with hyperlipidemia has a new order for colesevelam (Welchol). Which nursing action is appropriate when scheduling this medication? c. Give the patient's other medications 2 hours after colesevelam. d. Have the patient take the dose at the same time as the prescribed aspirin.

c. Give the patient's other medications 2 hours after colesevelam.

The nurse assesses a patient who has been hospitalized for 2 days. The patient has been receiving normal saline IV at 100 mL/hr, has a nasogastric tube to low suction, and is NPO. Which assessment finding would be a priority for the nurse to report to the health care provider? a. Oral temperature of 100.1°F b. Serum sodium level of 138 mEq/L (138 mmol/L) c. Gradually decreasing level of consciousness (LOC) d. Weight gain of 2 pounds (1 kg) over the admission weight

c. Gradually decreasing level of consciousness (LOC)

Heparin is ordered for a patient with a non-ST-segment-elevation myocardial infarction (NSTEMI). What is the purpose of the heparin? a. Heparin enhances platelet aggregation at the plaque site. b. Heparin decreases the size of the coronary artery plaque. c. Heparin prevents the development of new clots in the coronary arteries. d. Heparin dissolves clots that are blocking blood flow in the coronary arteries.

c. Heparin prevents the development of new clots in the coronary arteries.

Which assessment finding obtained by the nurse when assessing a patient with acute pericarditis should be reported immediately to the health care provider? a. Pulsus paradoxus 8 mm Hg b. Blood pressure (BP) of 168/94 mm Hg c. Jugular venous distention (JVD) to jaw level d. Level 6 (0 to 10 scale) chest pain with a deep breath

c. Jugular venous distention (JVD) to jaw level

A 21-yr-old woman is scheduled for percutaneous transluminal balloon valvuloplasty to treat mitral stenosis. Which information should the nurse include when explaining the advantages of valvuloplasty over valve replacement to the patient? a. Biologic valves will require immunosuppressive drugs after surgery. b. Mechanical mitral valves need to be replaced sooner than biologic valves. c. Lifelong anticoagulant therapy is needed after mechanical valve replacement. d. Ongoing cardiac care by a health care provider is not necessary after valvuloplasty.

c. Lifelong anticoagulant therapy is needed after mechanical valve replacement.

When caring for a patient on the first postoperative day after an abdominal aortic aneurysm repair, which assessment finding is most important for the nurse to communicate to the health care provider? a. Presence of flatus b. Hypoactive bowel sounds c. Maroon-colored liquid stool d. Abdominal pain with palpation

c. Maroon-colored liquid stool

A nurse is assessing a newly admitted patient with chronic heart failure who forgot to take prescribed medications and seems confused. The patient has peripheral edema and shortness of breath. Which assessment should the nurse complete first? a. Skin turgor b. Heart sounds c. Mental status d. Capillary refill

c. Mental status

A patient who is receiving dobutamine for the treatment of acute decompensated heart failure (ADHF) has the following nursing interventions included in the plan of care. Which action will be most appropriate for the registered nurse (RN) to delegate to an experienced licensed practical/vocational nurse (LPN/LVN)? a. Teach the patient the reasons for remaining on bed rest. b. Change the peripheral IV site according to agency policy. c. Monitor the patient's blood pressure and heart rate every hour. d. Titrate the rate to keep the systolic blood pressure >90 mm Hg.

c. Monitor the patient's blood pressure and heart rate every hour.

An older patient receiving iso-osmolar continuous tube feedings develops restlessness, agitation, and weakness. Which laboratory result should the nurse report to the health care provider immediately? a. K+ 3.4 mEq/L (3.4 mmol/L) b. Ca+2 7.8 mg/dL (1.95 mmol/L) c. Na+ 154 mEq/L (154 mmol/L) d. PO4-3 4.8 mg/dL (1.55 mmol/L)

c. Na+ 154 mEq/L (154 mmol/L)

Which assessment finding for a patient who has been admitted with a right calf venous thromboembolism (VTE) requires immediate action by the nurse? a. Erythema of right lower leg b. Complaint of right calf pain c. New onset shortness of breath d. Temperature of 100.4°F (38°C)

c. New onset shortness of breath

Which information about a patient who has been receiving thrombolytic therapy for an acute myocardial infarction is most important for the nurse to communicate to the health care provider? a. An increase in troponin levels from baseline b. A large bruise at the patient's IV insertion site c. No change in the patient's reported level of chest pain d. A decrease in ST-segment elevation on the electrocardiogram

c. No change in the patient's reported level of chest pain

A patient who is complaining of a "racing" heart and feeling "anxious" comes to the emergency department. The nurse places the patient on a heart monitor and obtains the following electrocardiographic (ECG) tracing. Which action should the nurse take next? a. Prepare to perform electrical cardioversion. b. Have the patient perform the Valsalva maneuver. c. Obtain the patient's vital signs including O2 saturation. d. Prepare to give a β-blocker medication to slow the heart rate.

c. Obtain the patient's vital signs including O2 saturation.

A patient recovering from heart surgery develops pericarditis and complains of level 6 (0 to 10 scale) chest pain with deep breathing. Which prescribed PRN medication will be the most appropriate for the nurse to give? a. Fentanyl 1 mg IV b. IV morphine sulfate 4 mg c. Oral ibuprofen (Motrin) 600 mg d. Oral acetaminophen (Tylenol) 650 mg

c. Oral ibuprofen (Motrin) 600 mg

A patient who has just been admitted with pulmonary edema is scheduled to receive the following medications. Which medication should the nurse question before giving? a. captopril 25 mg b. furosemide (Lasix) 60 mg c. digoxin (Lanoxin) 0.125 mg d. carvedilol (Coreg) 3.125 mg

d. carvedilol (Coreg) 3.125 mg

The nurse will plan discharge teaching about prophylactic antibiotics before dental procedures for which patient? a. Patient admitted with a large acute myocardial infarction b. Patient being discharged after an exacerbation of heart failure c. Patient who had a mitral valve replacement with a mechanical valve d. Patient being treated for rheumatic fever after a streptococcal infection

c. Patient who had a mitral valve replacement with a mechanical valve

After receiving change-of-shift report, which patient should the nurse assess first? a. Patient with serum potassium level of 5.0 mEq/L who is complaining of abdominal cramping b. Patient with serum sodium level of 145 mEq/L who has a dry mouth and is asking for a glass of water c. Patient with serum magnesium level of 1.1 mEq/L who has tremors and hyperactive deep tendon reflexes d. Patient with serum phosphorus level of 4.5 mg/dL who has multiple soft tissue calcium-phosphate precipitates

c. Patient with serum magnesium level of 1.1 mEq/L who has tremors and hyperactive deep tendon reflexes

A patient who was admitted with a myocardial infarction experiences a 45-second episode of ventricular tachycardia, then converts to sinus rhythm with a heart rate of 98 beats/min. Which action should the nurse take next? a. Immediately notify the health care provider. b. Document the rhythm and continue to monitor the patient. c. Prepare to give IV amiodarone per agency dysrhythmia protocol. d. Perform synchronized cardioversion per agency dysrhythmia protocol.

c. Prepare to give IV amiodarone per agency dysrhythmia protocol.

When caring for a patient with acute coronary syndrome who has returned to the coronary care unit after having angioplasty with stent placement, the nurse obtains the following assessment data. Which data indicate the need for immediate action by the nurse? a. Heart rate 102 beats/min c. Report of severe chest pain b. Pedal pulses 1+ bilaterally d. Blood pressure 103/54 mm Hg

c. Report of severe chest pain

During the admission process, the nurse obtains information about a patient through a physical assessment and diagnostic testing. Based on the data shown in the accompanying figure, which nursing diagnosis is appropriate? a. Deficient fluid volume b. Impaired gas exchange c. Risk for injury: seizures d. Risk for impaired skin integrity

c. Risk for injury: seizures

Which electrocardiographic (ECG) change is most important for the nurse to report to the health care provider when caring for a patient with chest pain? a. Inverted P wave c. ST-segment elevation b. Sinus tachycardia d. First-degree atrioventricular block

c. ST-segment elevation

A patient has recently started on digoxin (Lanoxin) in addition to furosemide (Lasix) and captopril for the management of heart failure. Which assessment finding by the home health nurse is a priority to communicate to the health care provider? a. Presence of 1+ to 2+ edema in the feet and ankles b. Palpable liver edge 2 cm below the ribs on the right side c. Serum potassium level 3.0 mEq/L after 1 week of therapy d. Weight increase from 120 pounds to 122 pounds over 3 days

c. Serum potassium level 3.0 mEq/L after 1 week of therapy

Which action by a new nurse who is giving fondaparinux (Arixtra) to a patient with a lower leg venous thromboembolism (VTE) indicates that more education about the drug is needed? a. The nurse avoids rubbing the injection site after giving the drug. b. The nurse injects the drug into the abdominal subcutaneous tissue. c. The nurse ejects the air bubble from the syringe before giving the drug. d. The nurse does not check partial thromboplastin time (PTT) before giving the drug.

c. The nurse ejects the air bubble from the syringe before giving the drug.

Which intervention by a new nurse who is caring for a patient who has just had an implantable cardioverter-defibrillator (ICD) inserted indicates a need for more teaching about the care of patients with ICDs? a. The nurse administers amiodarone (Cordarone) to the patient. b. The nurse helps the patient fill out the application for obtaining a Medic Alert device. c. The nurse encourages the patient to do active range of motion exercises for all extremities. d. The nurse teaches the patient that sexual activity can be resumed when the incision is healed.

c. The nurse encourages the patient to do active range of motion exercises for all extremities.

Which information will the nurse include when teaching a patient who is scheduled for a radiofrequency catheter ablation for treatment of atrial flutter? a. The procedure prevents or minimizes the risk for sudden cardiac death. b. The procedure uses cold therapy to stop the formation of the flutter waves. c. The procedure uses electrical energy to destroy areas of the conduction system. d. The procedure stimulates the growth of new conduction pathways between the atria.

c. The procedure uses electrical energy to destroy areas of the conduction system.

The nurse needs to quickly estimate the heart rate for a patient with a regular heart rhythm. Which method will be best to use? a. Count the number of large squares in the R-R interval and divide by 300. b. Print a 1-minute electrocardiogram (ECG) strip and count the number of QRS complexes. c. Use the 3-second markers to count the number of QRS complexes in 6 seconds and multiply by 10. d. Calculate the number of small squares between one QRS complex and the next and divide into 1500.

c. Use the 3-second markers to count the number of QRS complexes in 6 seconds and multiply by 10.

IV sodium nitroprusside is ordered for a patient with acute pulmonary edema. During the first hours of administration, the nurse will need to titrate the nitroprusside rate down if the patient develops a. ventricular ectopy. b. a dry, hacking cough. c. a systolic BP below 90 mm Hg. d. a heart rate below 50 beats/min.

c. a systolic BP below 90 mm Hg.

A patient with chronic heart failure who is taking a diuretic and an angiotensin-converting enzyme (ACE) inhibitor and who is on a low-sodium diet tells the home health nurse about a 5-lb weight gain in the past 3 days. The nurse's priority action will be to a. have the patient recall the dietary intake for the past 3 days. b. ask the patient about the use of the prescribed medications. c. assess the patient for clinical manifestations of acute heart failure. d. teach the patient about the importance of restricting dietary sodium.

c. assess the patient for clinical manifestations of acute heart failure.

While caring for a 23-yr-old patient with mitral valve prolapse (MVP) without valvular regurgitation, the nurse determines that discharge teaching has been effective when the patient states that it will be necessary to a. take antibiotics before any dental appointments. b. limit physical activity to avoid stressing the heart. c. avoid over-the-counter (OTC) drugs that contain stimulants. d. take an aspirin a day to prevent clots from forming on the valve.

c. avoid over-the-counter (OTC) drugs that contain stimulants.

Diltiazem (Cardizem) is ordered for a patient with newly diagnosed Prinzmetal's (variant) angina. When teaching the patient, the nurse will include the information that diltiazem will a. reduce heart palpitations. b. prevent coronary artery plaque. c. decrease coronary artery spasms. d. increase contractile force of the heart.

c. decrease coronary artery spasms.

While assessing a 68-yr-old with ascites, the nurse also notes jugular venous distention (JVD) with the head of the patient's bed elevated 45 degrees. The nurse knows this finding indicates a. decreased fluid volume. b. jugular vein atherosclerosis. c. increased right atrial pressure. d. incompetent jugular vein valves.

c. increased right atrial pressure.

During discharge teaching with an older patient who had a mitral valve replacement with a mechanical valve, the nurse must instruct the patient on the a. use of daily aspirin for anticoagulation. b. correct method for taking the radial pulse. c. need for frequent laboratory blood testing. d. need to avoid any physical activity for 1 month.

c. need for frequent laboratory blood testing.

The nurse plans discharge teaching for a patient with chronic heart failure who has prescriptions for digoxin (Lanoxin) and hydrochlorothiazide. Appropriate instructions for the patient include a. limit dietary sources of potassium. b. take the hydrochlorothiazide before bedtime. c. notify the health care provider if nausea develops. d. take the digoxin if the pulse is below 60 beats/min.

c. notify the health care provider if nausea develops

A patient who has chronic heart failure tells the nurse, "I was fine when I went to bed, but I woke up in the middle of the night feeling like I was suffocating!" The nurse will document this assessment finding as a. orthopnea. c. paroxysmal nocturnal dyspnea. b. pulsus alternans. d. acute bilateral pleural effusion.

c. paroxysmal nocturnal dyspnea.

A patient with diabetes mellitus and chronic stable angina has a new order for captopril . The nurse should teach the patient that the primary purpose of captopril is to a. decrease the heart rate. c. prevent changes in heart muscle. b. control blood glucose levels. d. reduce the frequency of chest pain.

c. prevent changes in heart muscle.

The nurse performing an assessment of a patient who has chronic peripheral artery disease (PAD) of the legs and an ulcer on the right second toe would expect to find a. dilated superficial veins. b. swollen, dry, scaly ankles. c. prolonged capillary refill in all the toes. d. serosanguineous drainage from the ulcer.

c. prolonged capillary refill in all the toes.

The nurse working on the heart failure unit knows that teaching an older female patient with newly diagnosed heart failure is effective when the patient states that a. she will take furosemide (Lasix) every day at bedtime. b. the nitroglycerin patch is to be used when chest pain develops. c. she will call the clinic if her weight goes up 3 pounds in 1 week. d. an additional pillow can help her sleep if she is short of breath at night.

c. she will call the clinic if her weight goes up 3 pounds in 1 week.

When caring for a patient with mitral valve stenosis, it is most important that the nurse assess for a. diastolic murmur. b. peripheral edema. c. shortness of breath on exertion. d. right upper quadrant tenderness.

c. shortness of breath on exertion.

When caring for a patient with infective endocarditis of the tricuspid valve, the nurse should monitor the patient for the development of a. flank pain. b. splenomegaly. c. shortness of breath. d. mental status changes.

c. shortness of breath.

A patient reports dizziness and shortness of breath for several days. During heart monitoring in the emergency department (ED), the nurse obtains the following electrocardiographic (ECG) tracing. The nurse interprets this heart rhythm as a. junctional escape rhythm. b. accelerated idioventricular rhythm. c. third-degree atrioventricular (AV) block. d. sinus rhythm with premature atrial contractions (PACs).

c. third-degree atrioventricular (AV) block.

While admitting an 82-yr-old patient with acute decompensated heart failure to the hospital, the nurse learns that the patient lives alone and sometimes confuses the "water pill" with the "heart pill." When planning for the patient's discharge the nurse will facilitate a a. plan for around-the-clock care. b. consultation with a psychologist. c. transfer to a long-term care facility. d. referral to a home health care agency.

c. transfer to a long-term care facility.

When the nurse is monitoring a patient who is undergoing exercise (stress) testing on a treadmill, which assessment finding requires the most rapid action by the nurse? a.Patient complaint of feeling tired b. Sinus tachycardia at a rate of 110 beats/min c. Inversion of T waves on the electrocardiogram d. Blood pressure (BP) increase from 134/68 to 150/80 mm Hg

c. Inversion of T waves on the electrocardiogram

A transesophageal echocardiogram (TEE) is ordered for a patient with possible endocarditis. Which action included in the standard TEE orders will the nurse need to accomplish first? a. Start an IV line. b. Start O2 per nasal cannula. c.Place the patient on NPO status. d.Give lorazepam (Ativan) 1 mg IV.

c.Place the patient on NPO status.

During the administration of the thrombolytic agent to a patient with an acute myocardial infarction, the nurse should stop the drug infusion if the patient experiences a. bleeding from the gums. b. increase in blood pressure. c. a decrease in level of consciousness. d. a nonsustained episode of ventricular tachycardia.

c.a decrease in level of consciousness.

The nurse obtains a health history from an older patient with a prosthetic mitral valve who has symptoms of infective endocarditis (IE). Which question by the nurse is most focused on identifying a risk factor for IE? a. "Do you have a history of a heart attack?" b. "Is there a family history of endocarditis?" c. "Have you had any recent immunizations?" d. "Have you had dental work done recently?"

d. "Have you had dental work done recently?"

After the nurse has finished teaching a patient about the use of sublingual nitroglycerin (Nitrostat), which patient statement indicates that the teaching has been effective? a. "I can expect nausea as a side effect of nitroglycerin." b. "I should only take nitroglycerin when I have chest pain." c. "Nitroglycerin helps prevent a clot from forming and blocking blood flow to my heart." d. "I will call an ambulance if I still have pain after taking three nitroglycerin 5 minutes apart."

d. "I will call an ambulance if I still have pain after taking three nitroglycerin 5 minutes apart."

Spironolactone (Aldactone), an aldosterone antagonist, is prescribed for a patient. Which statement by the patient indicates that the teaching about this medication has been effective? a. "I will try to drink at least 8 glasses of water every day." b. "I will use a salt substitute to decrease my sodium intake." c. "I will increase my intake of potassium-containing foods." d. "I will drink apple juice instead of orange juice for breakfast."

d. "I will drink apple juice instead of orange juice for breakfast."

Which statement made by a patient with coronary artery disease after the nurse has completed teaching about the therapeutic lifestyle changes (TLC) diet indicates that further teaching is needed? a. "I will switch from whole milk to 1% milk." b. "I like salmon and I will plan to eat it more often." c. "I can have a glass of wine with dinner if I want one." d. "I will miss being able to eat peanut butter sandwiches."

d. "I will miss being able to eat peanut butter sandwiches."

Which statement by a patient with restrictive cardiomyopathy indicates that the nurse's discharge teaching about self-management has been effective? a. "I will avoid taking aspirin or other antiinflammatory drugs." b. "I can restart my exercise program that includes hiking and biking." c. "I will need to limit my intake of salt and fluids even in hot weather." d. "I will take antibiotics before my teeth are cleaned at the dental office."

d. "I will take antibiotics before my teeth are cleaned at the dental office."

The nurse knows that discharge teaching about the management of a new permanent pacemaker has been most effective when the patient states a. "It will be several weeks before I can return to my usual activities." b. "I will avoid cooking with a microwave oven or being near one in use." c. "I will notify the airlines when I make a reservation that I have a pacemaker." d. "I won't lift the arm on the pacemaker side until I see the health care provider."

d. "I won't lift the arm on the pacemaker side until I see the health care provider."

Which information from a patient helps the nurse confirm the previous diagnosis of chronic stable angina? a. "The pain wakes me up at night." b. "The pain is level 3 to 5 (0 to 10 scale)." c. "The pain has gotten worse over the last week." d. "The pain goes away after a nitroglycerin tablet."

d. "The pain goes away after a nitroglycerin tablet."

The nurse is caring for a patient with mitral regurgitation. Referring to the figure below, where should the nurse listen to best hear a murmur typical of mitral regurgitation? a. 1 c. 3 b. 2 d. 4

d. 4

After receiving change-of-shift report about the following four patients on the cardiac care unit, which patient should the nurse assess first? a. A 39-yr-old patient with pericarditis who is complaining of sharp, stabbing chest pain b. A 56-yr-old patient with variant angina who is scheduled to receive nifedipine (Procardia) c. A 65-yr-old patient who had a myocardial infarction (MI) 4 days ago and is anxious about today's planned discharge d. A 59-yr-old patient with unstable angina who has just returned after a percutaneous coronary intervention (PCI)

d. A 59-yr-old patient with unstable angina who has just returned after a percutaneous coronary intervention (PCI)

The nurse has received change-of-shift report about the following patients on the progressive care unit. Which patient should the nurse see first? a. A patient with atrial fibrillation, rate 88 and irregular, who has a dose of warfarin (Coumadin) due b. A patient with second-degree atrioventricular (AV) block, type 1, rate 60, who is dizzy when ambulating c. A patient who is in a sinus rhythm, rate 98 and regular, recovering from an elective cardioversion 2 hours ago d. A patient whose implantable cardioverter-defibrillator (ICD) fired twice today and has a dose of amiodarone (Cordarone) due

d. A patient whose implantable cardioverter-defibrillator (ICD) fired twice today and has a dose of amiodarone (Cordarone) due

Which topic will the nurse plan to include in discharge teaching for a patient with heart failure with reduced ejection fraction (HFrEF)? a. Need to begin an aerobic exercise program several times weekly b. Use of salt substitutes to replace table salt when cooking and at the table c. Importance of making an annual appointment with the health care provider d. Benefits and side effects of angiotensin-converting enzyme (ACE) inhibitors

d. Benefits and side effects of angiotensin-converting enzyme (ACE) inhibitors

An outpatient who has chronic heart failure returns to the clinic after 2 weeks of therapy with metoprolol (Toprol XL). Which assessment finding is most important for the nurse to report to the health care provider? a. 2+ bilateral pedal edema b. Heart rate of 56 beats/min c. Complaints of increased fatigue d. Blood pressure (BP) of 88/42 mm Hg

d. Blood pressure (BP) of 88/42 mm Hg

A patient admitted to the coronary care unit (CCU) with an ST-segment-elevation myocardial infarction (STEMI) is restless and anxious. The blood pressure is 86/40 mm Hg, and heart rate is 132 beats/min. Based on this information, which nursing diagnosis is a priority for the patient? a. Acute pain related to myocardial infarction b. Anxiety related to perceived threat of death c. Stress overload related to acute change in health d. Decreased cardiac output related to cardiogenic shock

d. Decreased cardiac output related to cardiogenic shock

The nurse is caring for a patient who has a calcium level of 12.1 mg/dL. Which nursing action should the nurse include on the care plan? a. Maintain the patient on bed rest. b. Auscultate lung sounds every 4 hours. c. Monitor for Trousseau's and Chvostek's signs. d. Encourage fluid intake up to 4000 mL every day.

d. Encourage fluid intake up to 4000 mL every day.

Which nursing intervention for a patient who had an open repair of an abdominal aortic aneurysm 2 days previously is appropriate for the nurse to delegate to unlicensed assistive personnel (UAP)? a. Monitor the quality and presence of the pedal pulses. b. Teach the patient the signs of possible wound infection. c. Check the lower extremities for strength and movement. d. Help the patient to use a pillow to splint while coughing.

d. Help the patient to use a pillow to splint while coughing.

A patient is being evaluated for postthrombotic syndrome. Which assessment will the nurse perform? a. Ask about leg pain with exercise. b. Determine the ankle-brachial index. c. Assess capillary refill in the patient's toes. d. Inspect for presence of lipodermatosclerosis.

d. Inspect for presence of lipodermatosclerosis.

A patient admitted with acute dyspnea is newly diagnosed with dilated cardiomyopathy. Which information will the nurse plan to teach the patient about managing this disorder? a. A heart transplant should be scheduled as soon as possible. b. Elevating the legs above the heart will help relieve dyspnea. c. Careful compliance with diet and medications will prevent heart failure. d. Notify the health care provider about symptoms such as shortness of breath.

d. Notify the health care provider about symptoms such as shortness of breath.

The nurse is caring for a patient who is receiving IV furosemide (Lasix) and morphine for the treatment of acute decompensated heart failure (ADHF) with severe orthopnea. Which clinical finding is the best indicator that the treatment has been effective? a. Weight loss of 2 lb in 24 hours b. Hourly urine output greater than 60 mL c. Reduction in patient complaints of chest pain d. Reduced dyspnea with the head of bed at 30 degrees

d. Reduced dyspnea with the head of bed at 30 degrees

A patient had a non-ST-segment-elevation myocardial infarction (NSTEMI) 3 days ago. Which nursing intervention included in the plan of care is appropriate for the registered nurse (RN) to delegate to an experienced licensed practical/vocational nurse (LPN/LVN)? a. Evaluation of the patient's response to walking in the hallway b. Completion of the referral form for a home health nurse follow-up c. Education of the patient about the pathophysiology of heart disease d. Reinforcement of teaching about the purpose of prescribed medications

d. Reinforcement of teaching about the purpose of prescribed medications

A patient who was involved in a motor vehicle crash has had a tracheostomy placed to allow for continued mechanical ventilation. How should the nurse interpret the following arterial blood gas results: pH 7.48, PaO2 85 mm Hg, PaCO2 32 mm Hg, and HCO3 25 mEq/L? a. Metabolic acidosis c. Respiratory acidosis b. Metabolic alkalosis d. Respiratory alkalosis

d. Respiratory alkalosis

Which laboratory result for a patient with multifocal premature ventricular contractions (PVCs) is most important for the nurse to communicate to the health care provider? a. Blood glucose of 243 mg/dL c. Serum sodium of 134 mEq/L b. Serum chloride of 92 mEq/L d. Serum potassium of 2.9 mEq/L

d. Serum potassium of 2.9 mEq/L

A patient who is taking a potassium-wasting diuretic for treatment of hypertension complains of generalized weakness. Which action is appropriate for the nurse to take? a. Assess for facial muscle spasms. b. Ask the patient about loose stools. c. Recommend the patient avoid drinking orange juice with meals. d. Suggest that the health care provider order a basic metabolic panel.

d. Suggest that the health care provider order a basic metabolic panel.

The nurse is obtaining a health history from a 24-yr-old patient with hypertrophic cardiomyopathy (CMP). Which information obtained by the nurse is most important? a. The patient has a history of a recent upper respiratory infection. b. The patient has a family history of coronary artery disease (CAD). c. The patient reports using cocaine a "couple of times" as a teenager. d. The patient's 29-yr-old brother died from a sudden cardiac arrest.

d. The patient's 29-yr-old brother died from a sudden cardiac arrest.

Following an acute myocardial infarction, a previously healthy 63-yr-old develops clinical manifestations of heart failure. The nurse anticipates discharge teaching will include information about a. β-Adrenergic blockers. b. calcium channel blockers. c. digitalis and potassium therapy regimens. d. angiotensin-converting enzyme (ACE) inhibitors.

d. angiotensin-converting enzyme (ACE) inhibitors.

Several hours after a patient had an open surgical repair of an abdominal aortic aneurysm, the UAP reports to the nurse that urinary output for the past 2 hours has been 45 mL. The nurse notifies the health care provider and anticipates an order for a(n) a. hemoglobin count. c. serum creatinine level. b. additional antibiotic. d. increased IV infusion rate.

d. increased IV infusion rate.

An older patient with chronic atrial fibrillation develops sudden severe pain, pulselessness, pallor, and coolness in the right leg. The nurse should notify the health care provider and immediately a. apply a compression stocking to the leg. b. elevate the leg above the level of the heart. c. assist the patient in gently exercising the leg. d. keep the patient in bed in the supine position.

d. keep the patient in bed in the supine position.

A patient at the clinic says, "I always walk after dinner, but lately my leg cramps and hurts after just a few minutes of starting. The pain goes away after I stop walking, though." The nurse should a. look for the presence of tortuous veins bilaterally on the legs. b. ask about any skin color changes that occur in response to cold. c. assess for unilateral swelling, redness, and tenderness of either leg. d. palpate for the presence of dorsalis pedis and posterior tibial pulses.

d. palpate for the presence of dorsalis pedis and posterior tibial pulses.

Nadolol (Corgard) is prescribed for a patient with chronic stable angina and left ventricular dysfunction. To determine whether the drug is effective, the nurse will monitor for a. decreased blood pressure and heart rate. b. fewer complaints of having cold hands and feet. c. improvement in the strength of the distal pulses. d. participation in daily activities without chest pain.

d. participation in daily activities without chest pain.

The nurse has identified a nursing diagnosis of acute pain related to inflammatory process for a patient with acute pericarditis. An appropriate intervention by the nurse for this problem is to a. teach the patient to take deep, slow breaths to control the pain. b. force fluids to 3000 mL/day to decrease fever and inflammation. c. provide a fresh ice bag every hour for the patient to place on the chest. d. place the patient in Fowler's position, leaning forward on the overbed table.

d. place the patient in Fowler's position, leaning forward on the overbed table.

The nurse obtains a rhythm strip on a patient who has had a myocardial infarction and makes the following analysis: no visible P waves, PR interval not measurable, ventricular rate of 162, R-R interval regular, and QRS complex wide and distorted, and QRS duration of 0.18 second. The nurse interprets the patient's cardiac rhythm as a. atrial flutter. c. ventricular fibrillation. b. sinus tachycardia. d. ventricular tachycardia.

d. ventricular tachycardia.

The nurse teaches the patient being evaluated for rhythm disturbances with a Holter monitor to a.connect the recorder to a computer once daily. b. exercise more than usual while the monitor is in place. c. remove the electrodes when taking a shower or tub bath. d. keep a diary of daily activities while the monitor is worn.

d. keep a diary of daily activities while the monitor is worn.

The nurse is reviewing the 12-lead electrocardiograph (ECG) for a healthy 74-yr-old patient who is having an annual physical examination. What finding is of most concern to the nurse? A right bundle-branch block. B. The PR interval is 0.21 seconds. c.The QRS duration is 0.13 seconds. d.The heart rate (HR) is 41 beats/min.

d.The heart rate (HR) is 41 beats/min.


Related study sets

Life Policy Provisions, Riders and Options

View Set

Lab CE Course - Erythrocyte Inclusions

View Set

36. Portal vein, portosystemic (caval) shunts, and their medical importance.

View Set

prepu Chapter 17: Mood Disorders and Suicide

View Set

LESSON 8: OLFACTORY (I), OPTIC (II) AND OCULOMOTOR (III)

View Set